首页 大学物理 马文蔚 第五版 下册 第九章到第十一章课后答案

大学物理 马文蔚 第五版 下册 第九章到第十一章课后答案

举报
开通vip

大学物理 马文蔚 第五版 下册 第九章到第十一章课后答案第九章 振动 9-1 一个质点作简谐运动,振幅为A,在起始时刻质点的位移为,且向x 轴正方向运动,代表此简谐运动的旋转矢量为(  ) 题9-1 图 分析与解(b)图中旋转矢量的矢端在x 轴上投影点的位移为-A/2,且投影点的运动方向指向Ox 轴正向,即其速度的x分量大于零,故满足题意.因而正确答案为(b). 9-2 已知某简谐运动的振动曲线如图(a)所示,则此简谐运动的运动方程为(  ) 题9-2 图 分析与解 由振动曲线可知,初始时刻质点的位移为 –A/2,且向x 轴负方向运动.图(b)是其相应的旋转矢量图...

大学物理 马文蔚 第五版 下册 第九章到第十一章课后答案
第九章 振动 9-1 一个质点作简谐运动,振幅为A,在起始时刻质点的位移为,且向x 轴正方向运动,代 关于同志近三年现实表现材料材料类招标技术评分表图表与交易pdf视力表打印pdf用图表说话 pdf 此简谐运动的旋转矢量为(  ) 快递公司问题件快递公司问题件货款处理关于圆的周长面积重点题型关于解方程组的题及答案关于南海问题 9-1 图 分析 定性数据统计分析pdf销售业绩分析模板建筑结构震害分析销售进度分析表京东商城竞争战略分析 与解(b)图中旋转矢量的矢端在x 轴上投影点的位移为-A/2,且投影点的运动方向指向Ox 轴正向,即其速度的x分量大于零,故满足题意.因而正确答案为(b). 9-2 已知某简谐运动的振动曲线如图(a)所示,则此简谐运动的运动方程为(  ) 题9-2 图 分析与解 由振动曲线可知,初始时刻质点的位移为 –A/2,且向x 轴负方向运动.图(b)是其相应的旋转矢量图,由旋转矢量法可知初相位为.振动曲线上给出质点从–A/2 处运动到+A 处所需时间为1 s,由对应旋转矢量图可知相应的相位差,则角频率,故选(D).本题也可根据振动曲线所给信息,逐一代入方程来找出正确答案. 9-3  两个同周期简谐运动曲线如图(a) 所示, x1 的相位比x2 的相位(  ) (A) 落后    (B)超前  (C)落后    (D)超前 分析与解 由振动曲线图作出相应的旋转矢量图(b) 即可得到答案为(b). 题9-3 图 9-4 当质点以频率ν 作简谐运动时,它的动能的变化频率为(  ) (A)         (B)        (C)      (D) 分析与解 质点作简谐运动的动能表式为,可见其周期为简谐运动周期的一半,则频率为简谐运动频率ν的两倍.因而正确答案为(C). 9-5 图(a)中所画的是两个简谐运动的曲线,若这两个简谐运动可叠加,则合成的余弦振动的初相位为(  ) (A)         (B)        (C)      (D) 分析与解 由振动曲线可以知道,这是两个同振动方向、同频率简谐运动,它们的相位差是(即反相位).运动方程分别为和.它们的振幅不同.对于这样两个简谐运动,可用旋转矢量法,如图(b)很方便求得合运动方程为.因而正确答案为(D). 题9-5 图 9-6 有一个弹簧振子,振幅,周期,初相.试写出它的运动方程,并作出图、图和图. 题9-6 图 分析 弹簧振子的振动是简谐运动.振幅、初相、角频率是简谐运动方程的三个特征量.求运动方程就要设法确定这三个物理量.题中除、已知外,可通过关系式确定.振子运动的速度和加速度的计算仍与质点运动学中的计算方法相同. 解 因,则运动方程 根据题中给出的数据得 振子的速度和加速度分别为 、及图如图所示. 9-7 若简谐运动方程为,求:(1) 振幅、频率、角频率、周期和初相;(2)时的位移、速度和加速度. 分析 可采用比较法求解.将已知的简谐运动方程与简谐运动方程的一般形式作比较,即可求得各特征量.运用与上题相同的处理方法,写出位移、速度、加速度的表达式,代入值后,即可求得结果.     解 (1) 将与比较后可得:振幅A =0.10m,角频率,初相=0.25,则周期,频率. (2)时的位移、速度、加速度分别为 9-8 一远洋货轮,质量为m,浮在水面时其水平截面积为S.设在水面附近货轮的水平截面积近似相等,水的密度为ρ,且不计水的粘滞阻力,证明货轮在水中作振幅较小的竖直自由运动是简谐运动,并求振动周期. 分析 要证明货轮作简谐运动,需要分析货轮在平衡位置附近上下运动时,它所受的合外力与位移间的关系,如果满足,则货轮作简谐运动.通过即可求得振动周期. 证 货轮处于平衡状态时[图(a)],浮力大小为F =mg.当船上下作微小振动时,取货轮处于力平衡时的质心位置为坐标原点O,竖直向下为x 轴正向,如图(b)所示.则当货轮向下偏移x 位移时,受合外力为 其中为此时货轮所受浮力,其方向向上,大小为 题9-8 图 则货轮所受合外力为 式中是一常数.这表明货轮在其平衡位置上下所作的微小振动是简谐运动. 由可得货轮运动的微分方程为 令,可得其振动周期为 9-9 设地球是一个半径为R 的均匀球体,密度.现假定沿直径凿通一条隧道,若有一质量为m 的质点在此隧道内作无摩擦运动.(1) 证明此质点的运动是简谐运动;(2) 计算其周期. 题9-9 图 分析 证明方法与上题相似.分析质点在隧道内运动时的受力特征即可. 证 (1) 取图所示坐标.当质量为m 的质点位于x处时,它受地球的引力为 式中为引力常量,是以x 为半径的球体质量,即.令,则质点受力 因此,质点作简谐运动. (2) 质点振动的周期为 9-10 如图(a)所示,两个轻弹簧的劲度系数分别为、 .当物体在光滑斜面上振动时.(1) 证明其运动仍是简谐运动;(2) 求系统的振动频率. 题9-10 图 分析 从上两题的求解知道,要证明一个系统作简谐运动,首先要分析受力情况,然后看是否满足简谐运动的受力特征(或简谐运动微分方程).为此,建立如图(b)所示的坐标.设系统平衡时物体所在位置为坐标原点O,Ox 轴正向沿斜面向下,由受力分析可知,沿Ox 轴,物体受弹性力及重力分力的作用,其中弹性力是变力.利用串联时各弹簧受力相等,分析物体在任一位置时受力与位移的关系,即可证得物体作简谐运动,并可求出频率. 证 设物体平衡时两弹簧伸长分别为、,则由物体受力平衡,有                           (1) 按图(b)所取坐标,物体沿x 轴移动位移x时,两弹簧又分别被拉伸和,即.则物体受力为           (2) 将式(1)代入式(2)得                             (3) 由式(3)得、,而,则得到 式中为常数,则物体作简谐运动,振动频率   讨论 (1) 由本题的求证可知,斜面倾角θ 对弹簧是否作简谐运动以及振动的频率均不产生影响.事实上,无论弹簧水平放置、斜置还是竖直悬挂,物体均作简谐运动.而且可以证明它们的频率相同,均由弹簧振子的固有性质决定,这就是称为固有频率的原因.(2) 如果振动系统如图(c)(弹簧并联)或如图(d)所示,也可通过物体在某一位置的受力分析得出其作简谐运动,且振动频率均为,读者可以一试.通过这些例子可以知道,证明物体是否作简谐运动的思路是相同的. *9 -11 在如图(a)所示装置中,一劲度系数为k 的轻弹簧,一端固定在墙上,另一端连接一质量为的物体,置于光滑水平桌面上.现通过一质量m、半径为R 的定滑轮B(可视为匀质圆盘)用细绳连接另一质量为的物体C.设细绳不可伸长,且与滑轮间无相对滑动,求系统的振动角频率. 题9-11 图 分析 这是一个由弹簧、物体A、C 和滑轮B 组成的简谐运动系统.求解系统的振动频率可采用两种方法.(1) 从受力分析着手.如图(b)所示,设系统处于平衡状态时,与物体A 相连的弹簧一端所在位置为坐标原点O,此时弹簧已伸长,且.当弹簧沿轴正向从原点O 伸长x 时,分析物体A、C 及滑轮B的受力情况,并分别列出它们的动力学方程,可解得系统作简谐运动的微分方程.(2)从系统机械能守恒着手.列出系统机械能守恒方程,然后求得系统作简谐运动的微分方程. 解1 在图(b)的状态下,各物体受力如图(c)所示.其中.考虑到绳子不可伸长,对物体A、B、C 分别列方程,有                           (1)                             (2)                       (3)                               (4) 方程(3)中用到了、、及.联立式(1) ~式(4) 可得                       (5) 则系统振动的角频率为   解2 取整个振动装置和地球为研究系统,因没有外力和非保守内力作功,系统机械能守恒.设物体平衡时为初始状态,物体向右偏移距离x(此时速度为v、加速度为a)为末状态,则由机械能守恒定律,有 在列出上述方程时应注意势能(重力势能和弹性势能)零点的选取.为运算方便,选初始状态下物体C 所在位置为重力势能零点;弹簧原长时为弹性势能的零点.将上述方程对时间求导得 将,, 和 代入上式,可得                           (6) 式(6)与式(5)相同,表明两种解法结果一致. 9-12 一放置在水平桌面上的弹簧振子,振幅A=2.0 ×10-2 m,周期T=0.50s.当t=0 时,(1) 物体在正方向端点;(2) 物体在平衡位置、向负方向运动;(3) 物体在x =-1.0×10-2m 处, 向负方向运动; (4) 物体在x=-1.0×10-2 m处,向正方向运动.求以上各种情况的运动方程. 分析 在振幅A 和周期T 已知的条件下,确定初相φ是求解简谐运动方程的关键.初相的确定通常有两种方法.(1) 解析法:由振动方程出发,根据初始条件,即t =0 时,x =x0 和v =v0 来确定φ值.(2) 旋转矢量法:如图(a)所示,将质点P 在Ox 轴上振动的初始位置x0 和速度v0 的方向与旋转矢量图相对应来确定φ.旋转矢量法比较直观、方便,在分析中常采用. 题9-12 图 解 由题给条件知A =2.0 ×10-2 m,,而初相φ可采用分析中的两种不同方法来求. 解析法:根据简谐运动方程,当时有,.当(1)时,,则; (2)时,,,因,取; (3)时,, ,由,取; (4)时,, ,由,取. 旋转矢量法:分别画出四个不同初始状态的旋转矢量图,如图(b)所示,它们所对应的初相分别为,,,. 振幅A、角频率ω、初相φ均确定后,则各相应状态下的运动方程为 (1) (2) (3) (4) 9-13  有一弹簧, 当其下端挂一质量为m 的物体时, 伸长量为9.8 ×10-2 m.若使物体上、下振动,且 规定 关于下班后关闭电源的规定党章中关于入党时间的规定公务员考核规定下载规定办法文件下载宁波关于闷顶的规定 向下为正方向.(1) 当t =0 时,物体在平衡位置上方8.0 ×10-2 m 处,由静止开始向下运动,求运动方程.(2) 当t =0 时,物体在平衡位置并以0.6m·s-1的速度向上运动,求运动方程. 分析 求运动方程,也就是要确定振动的三个特征物理量A、ω和φ.其中振动的角频率是由弹簧振子系统的固有性质(振子质量m 及弹簧劲度系数k)决定的,即,k 可根据物体受力平衡时弹簧的伸长来计算;振幅A 和初相φ需要根据初始条件确定. 题9-13 图 解 物体受力平衡时,弹性力F 与重力P 的大小相等,即F =mg.而此时弹簧的伸长量Δl =9.8 ×10-2m.则弹簧的劲度系数k =F /Δl =mg /Δl.系统作简谐运动的角频率为   (1) 设系统平衡时,物体所在处为坐标原点,向下为x 轴正向.由初始条件t =0 时,x10 =8.0 ×10-2 m、v10 =0 可得振幅;应用旋转矢量法可确定初相[图(a)].则运动方程为   (2)t =0 时,x20 =0、v20 =0.6 m·s-1 ,同理可得;[图(b)].则运动方程为 9-14 某振动质点的x-t 曲线如图(a)所示,试求:(1) 运动方程;(2) 点P 对应的相位;(3) 到达点P 相应位置所需的时间. 分析 由已知运动方程画振动曲线和由振动曲线求运动方程是振动中常见的两类问题.本题就是要通过x -t 图线确定振动的三个特征量A、ω和,从而写出运动方程.曲线最大幅值即为振幅A;而ω、通常可通过旋转矢量法或解析法解出,一般采用旋转矢量法比较方便. 解 (1) 质点振动振幅A =0.10 m.而由振动曲线可画出t0 =0 和t1 =4 s时旋转矢量,如图(b) 所示.由图可见初相(或),而由得,则运动方程为 题9-14 图 (2) 图(a)中点P 的位置是质点从A/2 处运动到正向的端点处.对应的旋转矢量图如图(c) 所示.当初相取时,点P 的相位为(如果初相取成,则点P 相应的相位应表示为. (3) 由旋转矢量图可得,则. 9-15 作简谐运动的物体,由平衡位置向x 轴正方向运动,试问经过下列路程所需的最短时间各为周期的几分之几? (1) 由平衡位置到最大位移处;(2) 由平衡位置到x =A/2 处;  (3) 由x =A/2处到最大位移处. 解 采用旋转矢量法求解较为方便.按题意作如图所示的旋转矢量图,平衡位置在点O. (1) 平衡位置x1 到最大位移x3 处,图中的旋转矢量从位置1 转到位置3,故,则所需时间 (2) 从平衡位置x1 到x2 =A/2 处,图中旋转矢量从位置1转到位置2,故有,则所需时间 (3) 从x2 =A/2 运动到最大位移x3 处,图中旋转矢量从位置2 转到位置3,有,则所需时间 题9-15 图 9-16 在一块平板下装有弹簧,平板上放一质量为1.0 kg的重物.现使平板沿竖直方向作上下简谐运动,周期为0.50s,振幅为2.0×10-2 m.求:(1) 平板到最低点时,重物对平板的作用力;(2) 若频率不变,则平板以多大的振幅振动时,重物会跳离平板? (3) 若振幅不变,则平板以多大的频率振动时, 重物会跳离平板? 题9-16 图 分析 按题意作示意图如图所示.物体在平衡位置附近随板作简谐运动,其间受重力P 和板支持力FN 作用,FN 是一个变力.按牛顿定律,有                             (1)   由于物体是随板一起作简谐运动,因而有,则式(1)可改写为                       (2) (1) 根据板运动的位置,确定此刻振动的相位,由式(2)可求板与物体之间的作用力. (2) 由式(2)可知支持力 的值与振幅A、角频率ω和相位()有关.在振动过程中,当时最小.而重物恰好跳离平板的条件为=0,因此由式(2)可分别求出重物跳离平板所需的频率或振幅. 解 (1) 由分析可知,重物在最低点时,相位=0,物体受板的支持力为 重物对木块的作用力 与大小相等,方向相反. (2) 当频率不变时,设振幅变为A′.根据分析中所述,将=0及代入分析中式(2),可得   (3) 当振幅不变时,设频率变为.同样将=0及代入分析中式(2),可得 9-17 两质点作同频率、同振幅的简谐运动.第一个质点的运动方程为,当第一个质点自振动正方向回到平衡位置时,第二个质点恰在振动正方向的端点,试用旋转矢量图表示它们,并求第二个质点的运动方程及它们的相位差. 题9-17 图 解 图示为两质点在时刻t 的旋转矢量图,可见第一个质点M 的相位比第二个质点N 的相位超前,即它们的相位差Δφ=π/2.故第二个质点的运动方程应为 9-18 图(a)为一简谐运动质点的速度与时间的关系曲线,且振幅为2cm,求(1) 振动周期;(2) 加速度的最大值;(3) 运动方程. 分析 根据v-t 图可知速度的最大值vmax ,由vmax =Aω可求出角频率ω,进而可求出周期T 和加速度的最大值amax =Aω2 .在要求的简谐运动方程x =Acos(ωt +φ)中,因为A 和ω已得出,故只要求初相位φ即可.由v -t 曲线图可以知道,当t =0 时,质点运动速度v0 =vmax/2 =Aω/2,之后速度越来越大,因此可以判断出质点沿x 轴正向向着平衡点运动.利用v0 =-Aωsinφ就可求出φ. 解 (1) 由得,则 (2) (3) 从分析中已知,即 因为质点沿x 轴正向向平衡位置运动,则取,其旋转矢量图如图(b)所示.则运动方程为                题9-18 图 9-19 有一单摆,长为1.0m,最大摆角为5°,如图所示.(1) 求摆的角频率和周期;(2) 设开始时摆角最大,试写出此单摆的运动方程;(3) 摆角为3°时的角速度和摆球的线速度各为多少? 题9-19 图 分析 单摆在摆角较小时(θ<5°)的摆动,其角量θ与时间的关系可表示为简谐运动方程,其中角频率ω仍由该系统的性质(重力加速度g 和绳长l)决定,即.初相φ与摆角θ,质点的角速度与旋转矢量的角速度(角频率)均是不同的物理概念,必须注意区分. 解 (1) 单摆角频率及周期分别为 (2) 由时可得振动初相,则以角量表示的简谐运动方程为   (3) 摆角为3°时,有,则这时质点的角速度为 线速度的大小为   讨论 质点的线速度和角速度也可通过机械能守恒定律求解,但结果会有极微小的差别.这是因为在导出简谐运动方程时曾取,所以,单摆的简谐运动方程仅在θ 较小时成立. 9-20 为了测月球表面的重力加速度,宇航员将地球上的“秒摆”(周期为2.00s),拿到月球上去,如测得周期为4.90s,则月球表面的重力加速度约为多少? (取地球表面的重力加速度) 解 由单摆的周期公式可知,故有,则月球的重力加速度为 9-21 一飞轮质量为12kg,内缘半径r =0.6m,如图所示.为了测定其对质心轴的转动惯量,现让其绕内缘刃口摆动,在摆角较小时,测得周期为2.0s,试求其绕质心轴的转动惯量. 9-21 题图 分析 飞轮的运动相当于一个以刃口为转轴的复摆运动,复摆振动周期为,因此,只要知道复摆振动的周期和转轴到质心的距离,其以刃口为转轴的转动惯量即可求得.再根据平行轴定理,可求出其绕质心轴的转动惯量. 解 由复摆振动周期,可得.则由平行轴定理得 9-22 如图(a)所示,质量为1.0 ×10-2kg 的子弹,以500m·s-1的速度射入木块,并嵌在木块中,同时使弹簧压缩从而作简谐运动,设木块的质量为4.99 kg,弹簧的劲度系数为8.0 ×103 N·m-1 ,若以弹簧原长时物体所在处为坐标原点,向左为x 轴正向,求简谐运动方程. 题9-22 图 分析 可分为两个过程讨论.首先是子弹射入木块的过程,在此过程中,子弹和木块组成的系统满足动量守恒,因而可以确定它们共同运动的初速度v0 ,即振动的初速度.随后的过程是以子弹和木块为弹簧振子作简谐运动.它的角频率由振子质量m1 +m2 和弹簧的劲度系数k 确定,振幅和初相可根据初始条件(初速度v0 和初位移x0 )求得.初相位仍可用旋转矢量法求. 解 振动系统的角频率为 由动量守恒定律得振动的初始速度即子弹和木块的共同运动初速度v0 为 又因初始位移x0 =0,则振动系统的振幅为 图(b)给出了弹簧振子的旋转矢量图,从图中可知初相位,则简谐运动方程为 9-23 如图(a)所示,一劲度系数为k 的轻弹簧,其下挂有一质量为m1 的空盘.现有一质量为m2 的物体从盘上方高为h 处自由落入盘中,并和盘粘在一起振动.问:(1) 此时的振动周期与空盘作振动的周期有何不同? (2) 此时的振幅为多大? 题9-23 图 分析 原有空盘振动系统由于下落物体的加入,振子质量由m1 变为m1 + m2,因此新系统的角频率(或周期)要改变.由于,因此,确定初始速度v0 和初始位移x0 是求解振幅A 的关键.物体落到盘中,与盘作完全非弹性碰撞,由动量守恒定律可确定盘与物体的共同初速度v0 ,这也是该振动系统的初始速度.在确定初始时刻的位移x0 时,应注意新振动系统的平衡位置应是盘和物体悬挂在弹簧上的平衡位置.因此,本题中初始位移x0 ,也就是空盘时的平衡位置相对新系统的平衡位置的位移. 解 (1) 空盘时和物体落入盘中后的振动周期分别为 可见T′>T,即振动周期变大了. (2) 如图(b)所示,取新系统的平衡位置为坐标原点O.则根据分析中所述,初始位移为空盘时的平衡位置相对粘上物体后新系统平衡位置的位移,即 式中l1 =m1/k 为空盘静止时弹簧的伸长量,l2 =(m1 +m2)/k 为物体粘在盘上后,静止时弹簧的伸长量.由动量守恒定律可得振动系统的初始速度,即盘与物体相碰后的速度 式中是物体由h 高下落至盘时的速度.故系统振动的振幅为 本题也可用机械能守恒定律求振幅A. 9-24 如图所示,劲度系数为k 的轻弹簧,系一质量为m1 的物体,在水平面上作振幅为A的简谐运动.有一质量为m2 的粘土,从高度h 自由下落,正好在(a)物体通过平衡位置时,(b)物体在最大位移处时,落在物体上.分别求:(1)振动周期有何变化? (2)振幅有何变化? 题9-24图 分析 谐振子系统的周期只与弹簧的劲度系数和振子的质量有关.由于粘土落下前后,振子的质量发生了改变,因此,振动周期也将变化.至于粘土如何落下是不影响振动周期的.但是,粘土落下时将改变振动系统的初始状态,因此,对振幅是有影响的.在粘土落到物体上的两种不同情况中,系统在水平方向的动量都是守恒的.利用动量守恒定律可求出两种情况下系统的初始速度,从而利用机械能守恒定律(或公式)求得两种情况下的振幅. 解 (1) 由分析可知,在(a)、(b)两种情况中,粘土落下前后的周期均为 物体粘上粘土后的周期T′比原周期T 大. (2) (a) 设粘土落至物体前后,系统振动的振幅和物体经过平衡位置时的速度分别为A、v 和A′、v′.由动量守恒定律和机械能守恒定律可列出如下各式                             (1)                         (2)                             (3) 联立解上述三式,可得 即A′<A,表明增加粘土后,物体的振幅变小了. (b) 物体正好在最大位移处时,粘土落在物体上.则由动量守恒定律知它们水平方向的共同速度v′=m1v/(m1 +m2 ) =0,因而振幅不变,即 A′=A 9-25 质量为0.10kg的物体,以振幅1.0×10-2 m 作简谐运动,其最大加速度为4.0 m·s-1 求:(1) 振动的周期;(2) 物体通过平衡位置时的总能量与动能;(3) 物体在何处其动能和势能相等? (4) 当物体的位移大小为振幅的一半时,动能、势能各占总能量的多少? 分析 在简谐运动过程中,物体的最大加速度,由此可确定振动的周期T.另外,在简谐运动过程中机械能是守恒的,其中动能和势能互相交替转化,其总能量E =kA2/2.当动能与势能相等时,Ek =EP =kA2/4.因而可求解本题. 解 (1) 由分析可得振动周期   (2) 当物体处于平衡位置时,系统的势能为零,由机械能守恒可得系统的动能等于总能量,即   (3) 设振子在位移x0 处动能与势能相等,则有 得                        (4) 物体位移的大小为振幅的一半(即)时的势能为 则动能为                      9-26 一氢原子在分子中的振动可视为简谐运动.已知氢原子质量m =1.68 ×10-27 Kg,振动频率=1.0 ×1014 Hz,振幅A =1.0 ×10-11m.试计算:(1) 此氢原子的最大速度;(2) 与此振动相联系的能量. 解 (1) 简谐运动系统中振子运动的速度v =-Aωsin(ωt +φ),故氢原子 振动的最大速度为   (2) 氢原子的振动能量 9-27  质量m =10g 的小球与轻弹簧组成一振动系统, 按的规律作自由振动,求(1) 振动的角频率、周期、振幅和初相;(2) 振动的能量E;(3) 一个周期内的平均动能和平均势能. 解 (1) 将与比较后可得:角频率,振幅A =0.5cm,初相φ=π/3,则周期T =2π/ω=0.25 s (2) 简谐运动的能量 (3) 简谐运动的动能和势能分别为 则在一个周期中,动能与势能对时间的平均值分别为 9-28  已知两同方向、同频率的简谐运动的运动方程分别为;.求:(1) 合振动的振幅及初相;(2) 若有另一同方向、同频率的简谐运动,则为多少时,x1 +x3 的振幅最大? 又 为多少时,x2 +x3 的振幅最小? 题9-28 图 分析 可采用解析法或旋转矢量法求解.由旋转矢量合成可知,两个同方向、同频率简谐运动 的合成仍为一简谐运动,其角频率不变;合振动的振幅,其大小与两个分振动的初相差相关.而合振动的初相位 解 (1) 作两个简谐运动合成的旋转矢量图(如图).因为,故合振动振幅为 合振动初相位 (2) 要使x1 +x3 振幅最大,即两振动同相,则由得 要使x1 +x3 的振幅最小,即两振动反相,则由得 9-29 手电筒和屏幕质量均为m,且均被劲度系数为k 的轻弹簧悬挂于同一水平面上,如图所示.平衡时,手电筒的光恰好照在屏幕中心.设手电筒和屏幕相对于地面上下振动的表达式分别为和.试求在下述两种情况下,初相位φ1 、φ2 应满足的条件:(1) 光点在屏幕上相对于屏静止不动;(2) 光点在屏幕上相对于屏作振幅A′=2A的振动.并说明用何种方式起动,才能得到上述结果. 题9-29 图 分析 落在屏幕上的光点相对地面的运动和屏幕相对于地面的运动都已知道,且是两个简谐运动.因此由运动的合成不难写出光点相对屏的运动(实际上是两个同方向、同频率简谐运动的合成).根据相对运动公式,有 依题意 所以  可见光点对屏的运动就是两个同方向、同频率简谐运动和的合成.用与上题相同的方法即可求解本题.其中合运动振幅. 解 (1) 根据分析和参考上题求解,当要求任一时刻光点相对于屏不动,即,就是当时,即时(),A′=0.当光点相对于屏作振幅为2A的运动时,要求,即. (2) 由以上求解可知,要使光点相对于屏不动,就要求手电筒和屏的振动始终要同步,即同相位,为此,把它们往下拉A位移后,同时释放即可;同理,要使光点对屏作振幅为2A 的谐振动,两者必须相位相反,为此,让手电筒位于平衡点0 上方的-A处,而屏则位于+A 处同时释放,即可实现. 9-30 两个同频率的简谐运动1 和2 的振动曲线如图(a)所示,求(1)两简谐运动的运动方程x1 和x2;(2) 在同一图中画出两简谐运动的旋转矢量,并比较两振动的相位关系;(3) 若两简谐运动叠加,求合振动的运动方程. 分析 振动图已给出了两个简谐运动的振幅和周期,因此只要利用图中所给初始条件,由旋转矢量法或解析法求出初相位,便可得两个简谐运动的方程. 解 (1) 由振动曲线可知,A =0.1 m,T =2s,则ω=2π/T =πs-1 .曲线1表示质点初始时刻在x =0 处且向x 轴正向运动,因此φ1 =-π/2;曲线2 表示质点初始时刻在x =A /2 处且向x 轴负向运动,因此φ2 =π/3.它们的旋转矢量图如图(b)所示.则两振动的运动方程分别为  和  (2) 由图(b)可知振动2超前振动1 的相位为5π/6. (3) 其中 则合振动的运动方程为    题9-30 图 9-31 将频率为348 Hz的标准音叉振动和一待测频率的音叉振动合成,测得拍频为3.0Hz.若在待测频率音叉的一端加上一小块物体,则拍频数将减少,求待测音叉的固有频率. 分析 这是利用拍现象来测定振动频率的一种方法.在频率1 和拍频数Δ=|2 -1|已知的情况下,待测频率2 可取两个值,即2 =1 ±Δ.式中Δ前正、负号的选取应根据待测音叉系统质量改变时,拍频数变化的情况来决定. 解 根据分析可知,待测频率的可能值为 2 =1 ±Δ =(348 ±3) Hz 因振动系统的固有频率 ,即质量m 增加时,频率 减小.从题意知,当待测音叉质量增加时拍频减少,即|2 -1|变小.因此,在满足2 与Δ 均变小的情况下,式中只能取正号,故待测频率为2 =1 +Δ=351 Hz   *9-32 示波管的电子束受到两个互相垂直的电场的作用.电子在两个方向上的位移分别为 和,求在φ=0°,φ=30°,φ=90°各种情况下,电子在荧光屏上的轨迹方程. 解 这是两个振动方向互相垂直的同频率简谐运动的合成问题.合振动的轨迹方程为 式中A1 、A2 为两振动的振幅,Δφ为两个振动的初相差.本题中A1 =A2 =A,Δφ=φ,故有   (1) 当φ=0°时,有x =y,轨迹为一直线方程. (2) 当φ=30°时,有x2 +y2 - x y =A2/4,轨迹为椭圆方程. (3) 当φ=90°时,有x2 +y2 =A2 ,轨迹为圆方程.   *9-33 图示为测量液体阻尼系数的装置简图,将一质量为m 的物体挂在轻弹簧上,在空气中测得振动的频率为1 ,置于液体中测得的频率为2 ,求此系统的阻尼系数. 题9-33 图 分析 在阻尼不太大的情况下,阻尼振动的角频率ω与无阻尼时系统的固有角频率ω0 及阻尼系数δ 有关系式.因此根据题中测得的1 和2 (即已知ω0 、ω),就可求出δ. 解 物体在空气和液体中的角频率为 和,得阻尼系数为   *9-34 一弹簧振子系统,物体的质量m =1.0Kg,弹簧的劲度系数k =900N·m-1 .系统振动时受到阻尼作用,其阻尼系数δ =10.0s-1 .为了使振动持续,现另外加一周期性驱动外力.求:(1) 振子达到稳定时的振动角频率;(2) 若外力的角频率可以改变,当其值为多少时系统出现共振现象? 其共振的振幅多大? 分析 本题是物体在有阻尼条件下的受迫振动,其运动方程为                   (1) 等式右边第一项与阻尼有关,该项经一段时间后,因而消失.因此,稳定时系统的振动由第二项确定,它是简谐运动方程.其中为周期性外力的角频率,由此可知,此时振动的角频率即是周期性外力的角频率.而振幅为                         (2) 式中为周期性外界驱动力的力幅,是弹簧振子的固有角频率,m 是振子质量.当阻尼系数δ 一定时,振幅A 是外力的角频率ωP 的函数.共振时,振幅最大,故可采用对函数A(ωP)求极值的方法确定共振频率和振幅. 解 (1) 根据分析,受迫振动达到稳定时,系统作简谐运动的角频率即为周期性外力的角频率,故有   (2) 受迫振动达到稳定后, 其振幅.当时振幅将取得极大值,称共振现象.此时可解得周期性外界驱动力的角频率为 将上述结果代入振幅A 的表达式中,得共振时振幅为 9-35  在一个LC 振荡电路中, 若电容器两极板上的交变电压,电容,电路中的电阻可以忽略不计.求:(1) 振荡的周期;(2) 电路中的自感;(3) 电路中的电流随时间变化的规律. 分析 在不计电阻的前提下,该LC 电路是无阻尼自由振荡电路,在振荡过程中电容器两极板上的电压、电荷及电路中的电流均以相同的周期变化着.振荡周期为T =2π LC.因此,本题可通过已知的电压的角频率ω,求出振荡周期,然后可求出自感L.另外,电容器极板上电压U、电荷q 始终满足关系式q =CU.因此,在确定q =q(t)后,根据电流定义I =dq/dt,可求出电流的变化规律. 解 (1) 从题中已知的电压变化关系中得振荡周期为 (2) 由振荡电路周期得电路中的自感为 (3) 电路中电流随时间变化的规律为 9-36 用一个电容可在10.0 pF到360.0pF 范围内变化的电容器和一个自感线圈并联组成无线电收音机的调谐电路.(1) 该调谐电路可以接收的最大和最小频率之比是多少? (2) 为了使调谐频率能在5.0 ×105Hz 到1.5 ×106 Hz的频率范围内,需在原电容器上并联一个多大的电容? 此电路选用的自感应为多大? 分析 当自感L 一定时,要改变调谐频率的范围,只需改变电容的变化范围.本题采用并联电容C 的方法使电容由原有的变化范围Cmin ~Cmax 改变为Cmin +C~Cmax +C,从而达到新的调谐目的.为此,可根据,由原有电容比Cmax /Cmin 来确定对应的频率比.再由新要求的频率比来确定需要并联的电容的大小. 解 (1) 当线圈自感L 一定时,由,可得   (2) 为了在5.0 ×105 Hz ~1.5 ×106Hz 的频率范围内调谐,应满足 由此得在原电容器上需并联的电容为 此电路选用的线圈自感为 9-37 一振荡电路,已知C =0.25μF,L =1.015H.电路中电阻可忽略不计,电容器上电荷最大值为Q0 =2.5 ×10-6 C.(1) 写出电路接通后电容器两极板间的电势差随时间而变化的方程和电路中电流随时间而变化的方程;(2) 写出电场的能量、磁场能量及总能量随时间而变化的方程;(3) 求t1 =T/8 和t2 =T/4 时,电容器两极板间的电势差、电路中的电流、电场能、磁场能. 分析 无阻尼LC 振荡电路中电流、电容器极板上电荷以及电势差均以相同的频率随时间作正弦或余弦变化.如果令极板上电荷,则由 可得电路中的电流I、极板两端电势差U 的变化规律.利用电磁场中电场能量和磁场能量公式可写出它们随时间t 的函数关系式和特定时刻的瞬时值. 解 (1) LC无阻尼振荡电路的振荡角频率为 若以电路闭合的瞬间为计时起点,此时极板上电荷最大.则任一时刻极板上的电荷为 该时刻电路中的电流为 极板两端电势差为 (2) 任意时刻电场能量、磁场能量及总能量分别为 (3) 由,可得T =0.001 s,则当t1 =T/8 时,由上述各式可得 同理,当时可得 由上述结果可以看出LC 电路在无阻尼振荡过程中,总的电磁场能量是不变的,即满足能量守恒定律. 第十章 波  动 10-1 图(a)表示t =0 时的简谐波的波形图,波沿x 轴正方向传播,图(b)为一质点的振动曲线.则图(a)中所表示的x =0 处振动的初相位与图(b)所表示的振动的初相位分别为(  ) 题10-1 图 (A) 均为零     (B) 均为      (C) 均为 (D) 与    (E) 与 分析与解 本题给了两个很相似的曲线图,但本质却完全不同.求解本题要弄清振动图和波形图不同的物理意义.图(a)描述的是连续介质中沿波线上许许多多质点振动在t 时刻的位移状态.其中原点处质点位移为零,其运动方向由图中波形状态和波的传播方向可以知道是沿y 轴负向,利用旋转矢量法可以方便的求出该质点振动的初相位为π/2.而图(b)是一个质点的振动曲线图,该质点在t =0 时位移为0,t >0 时,由曲线形状可知,质点向y 轴正向运动,故由旋转矢量法可判知初相位为-π/2,答案为(D). 10-2 机械波的表达式为,则(  ) (A) 波长为100 m        (B) 波速为10 m·s-1 (C) 周期为1/3 s                (D) 波沿x 轴正方向传播 分析与解 波动方程的一般表式为,其中A 为振幅,φ为初相,u 为波速.x/u 前的“-”表示波沿x 轴正向传播,“+”表示波沿x轴负向传播.因此将原式写为和一般式比较可知(B)、(D) 均不对.而由ω=2π/T =6πs-1 可知T =(1/3)s.则λ=uT =33.3 m,因此(A)也不对.只有(C)正确. 10-3 一平面简谐波,沿x 轴负方向传播,角频率为ω,波速为u.设时刻的波形如图(a)所示,则该波的表达式为(  ) 题10-3 图 分析与解 因为波沿x 轴负向传播,由上题分析知(A)、(B)表式不正确.找出(C)、(D)哪个是正确答案,可以有很多方法.这里给出两个常用方法.方法一:直接将t =T/4,x=0 代入方程,那么对(C)有y0 =A、对(D)有y0 =0,可见(D)的结果与图一致.方法二:用旋转矢量法求出波动方程的初相位.由图(a)可以知道t =T/4 时原点处质点的位移为0,且向y 轴正向运动,则此时刻的旋转矢量图如图(b)所示.要求初相位,只要将该时刻的旋转矢量反转(顺时针转)Δφ=ω·Δt =ω·T/4  =π/2,如图(b)所示,即得φ0 =π.同样得(D)是正确答案. 题10-4 图 10-4 如图所示,两列波长为λ的相干波在点P 相遇.波在点S1 振动的初相是φ1 ,点S1 到点P的距离是r1 .波在点S2的初相是φ2 ,点S2 到点P 的距离是r2 ,以k 代表零或正、负整数,则点P 是干涉极大的条件为(  ) 分析与解 P 是干涉极大的条件为两分振动的相位差,而两列波传到P 点时的两分振动相位差为,故选项(D)正确. 10-5 在驻波中,两个相邻波节间各质点的振动(  ) (A) 振幅相同,相位相同      (B) 振幅不同,相位相同 (C) 振幅相同,相位不同            (D) 振幅不同,相位不同 分析与解 驻波方程为,因此根据其特点,两波节间各点运动同相位,但振幅不同.因此正确答案为(B). 10-6 频率为=1.25 ×104 Hz 的平面简谐纵波沿细长的金属棒传播,棒的弹性模量为E =1.90 ×1011 N·m -2 ,棒的密度ρ =7.6 ×103 Kg·m -3 .求该纵波的波长. 分析 因机械波传播速度与介质性质有关,固体中纵波传播速度.而波的特征量波长λ与波速u、频率之间有λ=u/.所以,频率一定的振动在不同介质中传播时,其波长不同.由上述关系可求得波长. 解 由分析可知金属棒中传播的纵波速度,因此,该纵波的波长为 10-7 一横波在沿绳子传播时的波动方程为.(1) 求波的振幅、波速、频率及波长;(2) 求绳上质点振动时的最大速度;(3) 分别画出t =1s 和t =2 s时的波形,并指出波峰和波谷.画出x =1.0 m处质点的振动曲线并讨论其与波形图的不同. 分析 (1) 已知波动方程(又称波函数)求波动的特征量(波速u、频率、振幅A 及波长λ等),通常采用比较法.将已知的波动方程按波动方程的一般形式 关于书的成语关于读书的排比句社区图书漂流公约怎么写关于读书的小报汉书pdf 写,然后通过比较确定各特征量(式中前“-”、“+”的选取分别对应波沿x 轴正向和负向传播).比较法思路清晰、求解简便,是一种常用的解题方法.(2) 讨论波动问题,要理解振动物理量与波动物理量之间的内在联系与区别.例如区分质点的振动速度与波速的不同,振动速度是质点的运动速度,即v =dy/dt;而波速是波线上质点运动状态的传播速度(也称相位的传播速度、波形的传播速度或能量的传播速度),其大小由介质 的性质决定.介质不变,波速保持恒定.(3) 将不同时刻的t 值代入已知波动方程,便可以得到不同时刻的波形方程y =y(x),从而作出波形图.而将确定的x 值代入波动方程,便可以得到该位置处质点的运动方程y =y(t),从而作出振动图. 解 (1) 将已知波动方程表示为 与一般表达式比较,可得 则                      (2) 绳上质点的振动速度 则                              (3) t =1s 和t =2s  时的波形方程分别为 波形图如图(a)所示. x =1.0m 处质点的运动方程为 振动图线如图(b)所示. 波形图与振动图虽在图形上相似,但却有着本质的区别.前者表示某确定时刻波线上所有质点的位移情况,而后者则表示某确定位置的一个质点,其位移随时间变化的情况. 题10-7 图 10-8 波源作简谐运动,其运动方程为,它所形成的波形以30m·s-1 的速度沿一直线传播.(1) 求波的周期及波长;(2) 写出波动方程. 分析 已知波源运动方程求波动物理量及波动方程,可先将运动方程与其一般形式进行比较,求出振幅A、角频率ω及初相φ0 ,而这三个物理量与波动方程的一般形式中相应的三个物理量是相同的.再利用题中已知的波速u 及公式ω=2πν =2π/T 和λ=u T 即可求解. 解 (1) 由已知的运动方程可知,质点振动的角频率.根据分析中所述,波的周期就是振动的周期,故有 波长为 λ=uT =0.25 m (2) 将已知的波源运动方程与简谐运动方程的一般形式比较后可得A =4.0 ×10-3m,,φ0 =0故以波源为原点,沿x 轴正向传播的波的波动方程为 10-9 已知一波动方程为.(1) 求波长、频率、波速和周期;(2) 说明x =0 时方程的意义,并作图表示. 题10-9 图 分析 采用比较法.将题给的波动方程改写成波动方程的余弦函数形式,比较可得角频率ω、波速u,从而求出波长、频率等.当x 确定时波动方程即为质点的运动方程y =y(t). 解 (1) 将题给的波动方程改写为与比较后可得波速u =15.7 m·s-1 , 角频率ω=10πs-1 ,故有 (2) 由分析知x =0 时,方程表示位于坐标原点的质点的运动方程(如图). 10-10 波源作简谐运动,周期为0.02s,若该振动以100m·s-1 的速度沿直线传播,设t =0时,波源处的质点经平衡位置向正方向运动,求:(1) 距波源15.0m 和5.0 m 两处质点的运动方程和初相;(2) 距波源为16.0 m 和17.0m的两质点间的相位差. 分析 (1) 根据题意先设法写出波动方程,然后代入确定点处的坐标,即得到质点的运动方程.并可求得振动的初相.(2) 波的传播也可以看成是相位的传播.由波长λ的物理含意,可知波线上任两点间的相位差为Δφ=2πΔx/λ. 解 (1) 由题给条件,可得 当t =0 时,波源质点经平衡位置向正方向运动,因而由旋转矢量法可得该质点的初相为φ0 =-π/2(或3π/2).若以波源为坐标原点,则波动方程为 距波源为x1 =15.0 m 和x2 =5.0 m 处质点的运动方程分别为 它们的初相分别为φ10 =-15.5π和φ10 =-5.5π(若波源初相取φ0=3π/2,则初相φ10 =-13.5π,φ10 =-3.5π.) (2) 距波源16.0m 和17.0 m 两点间的相位差 10-11 有一平面简谐波在空间传播.已知在波线上某点B 的运动规律为,就图(a)(b)(c)给出的三种坐标取法,分别列出波动方程.并 用这三个方程来描述与B 相距为b 的P 点的运动规律. 分析 (1) 波动方程的一般表式为,式中振幅A、角频率ω和波速u 从B 点运动方程和所给图均已知.因此只要求出原点的初相φ0 .而对(a)、(b)情况,B点即为原点,所以φ0=φ,对情况(c),原点比B 点超前相位Δφ=ω1 /u,则φ0 =φ+ω1/u.(2) 写出三种情况下波动方程后只要将P 点相应的坐标代入即可写出P 点的运动规律. 解 (1) 根据分析和图示波的传播方向,有 (a) 情况下: (b) 情况下: (c) 情况下: 题10-11图 (2) 将P 点的x 坐标值分别代入上述相应的波动方程可得三种情况下均有: 讨论 由于三种情况下,在沿波传播方向上,P 点均落在B 点后距离为b处,即P点的振动均比B 点的振动落后时间b/u,落后相位ωb/u,因而P点的运动方程均为. 10-12  图示为平面简谐波在t =0 时的波形图,设此简谐波的频率为250Hz,且此时图中质点P 的运动方向向上.求:(1) 该波的波动方程;(2) 在距原点O 为7.5 m 处质点的运动方程与t =0 时该点的振动速度. 分析 (1) 从波形曲线图获取波的特征量,从而写出波动方程是建立波动方程的又一途径.具体步骤为:1. 从波形图得出波长λ、振幅A 和波速u =λ;2. 根据点P 的运动趋势来判断波的传播方向,从而可确定原点处质点的运动趋向,并利用旋转矢量法确定其初相φ0 .(2) 在波动方程确定后,即可得到波线上距原点O 为x 处的运动方程y =y(t),及该质点的振动速度=dy/dt. 解 (1) 从图中得知,波的振幅A=0.10 m,波长λ=20.0m,则波速u =λ=5.0 ×103 m·s-1 .根据t =0 时点P 向上运动,可知波沿Ox 轴负向传播,并判定此时位于原点处的质点将沿Oy 轴负方向运动.利用旋转矢量法可得其初相φ0 =π/3.故波动方程为 (2) 距原点O 为x =7.5m 处质点的运动方程为 t =0 时该点的振动速度为 题10-12 图 10-13 如图所示为一平面简谐波在t =0 时刻的波形图,求(1)该波的波动方程;(2) P 处质点的运动方程. 题10-13 图 分析 (1) 根据波形图可得到波的波长λ、振幅A 和波速u,因此只要求初相φ,即可写出波动方程.而由图可知t =0 时,x =0 处质点在平衡位置处,且由波的传播方向可以判断出该质点向y 轴正向运动,利用旋转矢量法可知φ=-π/2.(2) 波动方程确定后,将P 处质点的坐标x 代入波动方程即可求出其运 动方程yP =yP(t). 解 (1) 由图可知振幅A =0.04 m, 波长λ=0.40 m, 波速u =0.08m·s-1 ,则ω=2π/T =2πu/λ=(2π/5)s-1 ,根据分析已知φ=-π/2,因此波动方程为 (2) 距原点O 为x =0.20m 处的P 点运动方程为 10-14 一平面简谐波,波长为12 m,沿Ox 轴负向传播.图(a)所示为x =1.0 m 处质点的振动曲线,求此波的波动方程. 题10-14图 分析 该题可利用振动曲线来获取波动的特征量,从而建立波动方程.求解的关键是如何根据图(a) 写出它所对应的运动方程.较简便的方法是旋转矢量法. 解 由图(a)可知质点振动的振幅A =0.40 m,t =0 时位于x =1.0 m处的质点在A/2 处并向Oy 轴正向移动.据此作出相应的旋转矢量图(b),从图中可知.又由图(a)可知,t =5 s 时,质点第一次回到平衡位置,由图(b)可看出ωt =5π/6,因而得角频率ω=(π/6) s-1 .由上述特征量可写出x =1.0 m处质点的运动方程为 将波速及x=1.0 m 代入波动方程的一般形式中,并与上述x =1.0 m 处的运动方程作比较,可得φ0 =-π/2,则波动方程为 10-15 图中(Ⅰ)是t =0 时的波形图,(Ⅱ)是t =0.1 s 时的波形图,已知T >0.1 s,写出波动方程的表达式. 题10-15 图 分析 已知波动方程的形式为 从如图所示的t =0 时的波形曲线Ⅰ,可知波的振幅A 和波长λ,利用旋转矢量法可确定原点处质点的初相φ0 .因此,确定波的周期就成为了解题的关键.从题给条件来看,周期T 只能从两个不同时刻的波形曲线之间的联系来得到.为此,可以从下面两个不同的角度来分析. (1) 由曲线(Ⅰ)可知,在t =0 时,原点处的质点处在平衡位置且向Oy 轴负向运动,而曲线(Ⅱ)则表明,经过0.1s 后,该质点已运动到Oy轴上的-A 处.因此,可列方程kT+T/4 =0.1s,在一般情形下,k =0,1,2,…这就是说,质点在0.1s 内,可以经历k 个周期振动后再回到-A 处,故有T =0.1/(k +0.25) s. (2) 从波形的移动来分析.因波沿Ox 轴正方向传播,波形曲线(Ⅱ)可视为曲线(Ⅰ)向右平移了Δx =uΔt =λΔt/T.由图可知,Δx =kλ+λ/4,故有kλ+λ/4 =λΔt/T,同样也得T =0.1/(k +0.25)s.应当注意,k 的取值由题给条件T >0.1 s所决定. 解 从图中可知波长λ=2.0 m,振幅A =0.10 m.由波形曲线(Ⅰ)得知在t =0 时,原点处质点位于平衡位置且向Oy 轴负向运动,利用旋转矢量法可得φ0 =π/2.根据上面的分析,周期为 由题意知T >0.1 s,故上式成立的条件为k =0,可得T =0.4 s.这样,波动方程可写成 10-16 平面简谐波的波动方程为. 求:(1) t =2.1 s 时波源及距波源0.10m 两处的相位;(2) 离波源0.80 m及0.30 m 两处的相位差. 解 (1) 将t =2.1 s和x =0 代入题给波动方程,可得波源处的相位 将t =2.1 s 和x′=0.10 m 代入题给波动方程,得0.10 m 处的相位为 (2) 从波动方程可知波长λ=1.0 m.这样,x1 =0.80 m 与x2 =0.30 m两点间的相位差 10-17 为了保持波源的振动不变,需要消耗4.0 W 的功率.若波源发出的是球面波(设介质不吸收波的能量).求距离波源5.0 m和10.0 m处的能流密度. 分析 波的传播伴随着能量的传播.由于波源在单位时间内提供的能量恒定,且介质不吸收能量,故对于球面波而言,单位时间内通过任意半径的球面的能量(即平均能流)相同,都等于波源消耗的功率.而在同一个球面上各处的能流密度相同,因此,可求出不同位置的能流密度I =/S. 解 由分析可知,半径r 处的能流密度为 当r1 =5.0 m、r2 =10.0m 时,分别有   10-18 有一波在介质中传播,其波速u =1.0 ×103m·s-1 ,振幅A =1.0 ×10-4 m,频率ν =1.0 ×103Hz.若介质的密度为ρ =8.0×102 kg·m-3 ,求:(1) 该波的能流密度;(2) 1 min 内垂直通过4.0 ×10-4m2 的总能量. 解 (1) 由能流密度I 的表达式得 (2) 在时间间隔Δt =60 s 内垂直通过面积S 的能量为 10-19 如图所示,两振动方向相同的平面简谐波波源分别位于A、B 两点.设它们相位相同,且频率均为=30Hz,波速u =0.50 m·s-1 .求在P 点处两列波的相位差. 分析 在均匀介质中,两列波相遇时的相位差Δφ一般由两部分组成,即它们的初相差φA -φB 和由它们的波程差而引起的相位差2πΔr/λ.本题因φA=φB ,故它们的相位差只取决于波程差. 解 在图中的直角三角形ABP 中 两列波在点P 处的波程差为Δr =AP -BP,则相位差为 题10-19图 10-20 如图所示,两相干波源分别在P、Q 两点处,它们发出频率为ν、波长为λ,初相相同的两列相干波.设PQ =3λ/2,R 为PQ 连线上的一点.求:(1) 自P、Q 发出的两列波在R处的相位差;(2) 两波在R 处干涉时的合振幅. 题10-20 图 分析 因两波源的初相相同,两列波在点R 处的相位差Δφ仍与上题一样,由它们的波程差决定.因R 处质点同时受两列相干波的作用,其振动为这两个同频率、同振动方向的简谐运动的合成,合振幅. 解 (1) 两列波在R 处的相位差为 (2) 由于,则合振幅为 10-21 两相干波波源位于同一介质中的A、B 两点,如图(a)所示.其振幅相等、频率皆为100 Hz,B 比A 的相位超前π.若A、B 相距30.0m,波速为u =400 m·s-1 ,试求AB 连线上因干涉而静止的各点的位置. 题10-21 图 分析 两列相干波相遇时的相位差.因此,两列振幅相同的相干波因干涉而静止的点的位置,可根据相消条件获得. 解 以A、B两点的中点O 为原点,取坐标如图(b)所示.两波的波长均为λ=u/=4.0 m.在A、B 连线上可分三个部分进行讨论. 1.  位于点A 左侧部分 因该范围内两列波相位差恒为2π的整数倍,故干涉后质点振动处处加强,没有静止的点. 2.  位于点B 右侧部分 显然该范围内质点振动也都是加强,无干涉静止的点. 3.  在A、B两点的连线间,设任意一点P 距原点为x.因,,则两列波在点P 的相位差为 根据分析中所述,干涉静止的点应满足方程 得                          因x≤15 m,故k≤7.即在A、B之间的连线上共有15 个静止点. 10-22  图(a)是干涉型消声器结构的原理图,利用这一结构可以消除噪声.当发动机排气噪声声波经管道到达点A 时,分成两路而在点B 相遇,声波因干涉而相消.如果要消除频率为300 Hz 的发动机排气噪声,则图中弯管与直管的长度差Δr =r2 -r1 至少应为多少? (设声波速度为340 m·s-1 ) 题10-22 图 分析 一列声波被分成两束后再相遇,将形成波的干涉现象.由干涉相消条件,可确定所需的波程差,即两管的长度差Δr. 解 由分析可知,声波从点A 分开到点B 相遇,两列波的波程差Δr =r2 - r1 ,故它们的相位差为 由相消静止条件Δφ=(2k +1)π,(k =0,±1,±2,…) 得                          Δr =(2k +1)λ/2 根据题中要求令k =0 得Δr 至少应为 讨论 在实际应用中,由于噪声是由多种频率的声波混合而成,因而常将具有不同Δr 的消声单元串接起来以增加消除噪声的能力.图(b)为安装在摩托车排气系统中的干涉消声器的结构原理图. 10-23 如图所示,x =0 处有一运动方程为的平面波波源,产生的波沿x 轴正、负方向传播.MN 为波密介质的反射面,距波源3λ/4.求:(1) 波源所发射的波沿波源O 左右传播的波动方程;(2) 在MN 处反射波的波动方程;(3) 在O ~MN 区域内形成的驻波方程,以及波节和波腹的位置;(4) x >0区域内合成波的波动方程. 题10-23 图 分析 知道波源O 点的运动方程,可以写出波沿x 轴负向和正向传播的方程分别为和.因此可以写出y1 在MN 反射面上P 点的运动方程.设反射波为y3 ,它和y1 应是同振动方向、同振幅、同频率的波,但是由于半波损失,它在P 点引起的振动和y1 在P 点引起的振动反相.利用y1 在P 点的运动方程可求y3 在P 点的运动方程,从而写出反射波y3 .在O ~MN 区域由y1 和Y3 两列同频率、同振动方向、同振幅沿相反方向传播的波合成形成驻波.在x >0区域是同传播方向的y2 和y3 合成新的行波. 解 (1) 由分析已知:沿左方向和右方向传播的波动方程分别为 和 (2) y1 在反射面MN 处引起质点P 振动的运动方程 因半波损失反射波y3 在此处引起的振动为 设反射波的波动方程为,则反射波在x =-3λ/4处引起的振动为 与上式比较得,故反射波的波动方程为 (3) 在O ~MN 区域由y1 和y3 合成的驻波y4 为 波节的位置:,取k =-1, -2,即x =-λ/4, -3λ/4 处为波节. 波腹的位置:,取k =0,-1,即x =0,-λ/2 处为波腹. (4) 在x >0 区域,由y2 和y3 合成的波y5 为 这表明:x >0 区域内的合成波是振幅为2A 的平面简谐波. 10-24 一弦上的驻波方程式为 (1) 若将此驻波看成是由传播方向相反,振幅及波速均相同的两列相干波叠加而成的,求它们的振幅及波速;(2) 求相邻波节之间的距离;(3) 求t =3.0 ×10-3 s 时位于x=0.625 m 处质点的振动速度. 分析 (1) 采用比较法.将本题所给的驻波方程,与驻波方程的一般形式相比较即可求得振幅、波速等.(2) 由波节位置的表达式可得相邻波节的距离.(3) 质点的振动速度可按速度定义v =dy/dt 求得. 解 (1) 将已知驻波方程    与驻波方程的一般形式作比较,可得两列波的振幅A =1.5 ×10-2 m,波长λ=1.25 m,频率=275 Hz,则波速u =λ=343.8m·s-1 . (2) 相邻波节间的距离为 (3) 在t =3.0 ×10-3 s 时,位于x=0.625 m 处质点的振动速度为 *10 -25 在下述两种情况下,求长度为0.15 m 的风琴管的基频和前四个谐频.(1) 管子两端开口;(2) 管子的一端封闭,一端开口.设声速为340m·s-1 . 分析 当风琴管的某个端口封闭时,那么风琴管内形成的驻波在该端口就是波节.而当风琴管的端口开口时,就形成波腹.根据限定区域内驻波形成条件(如图所示),当管子两端为波腹时,其管长与波长有关系式L =kλk /2 成立,k 为正整数.而当管子一端为波节、另一端为波腹时,管长与波长有关系式L =(2k -1)λk /4 成立.可见取不同的k 值,得到不同的λk ,管内就出现不同频率k 的波.对应k =1 称为基频,k =2,3,4,…称为各次谐频. 题10-25 图 解 (1) 根据分析由L =kλk /2 和νk =u /λk 可得 k =ku/2L (k =1,2,3,…) 因此,基频: 1 =1133 Hz 二次谐频:2 =2267 Hz 三次谐频:3 =3400 Hz 四次谐频:4 =4533 Hz 五次谐频:5 =5667 Hz (2) 同样根据分析由L =(2k -1)λk /4  和νk =u /λk 可得 k =(2k -1)u /4L (k =1,2,3,…) 因此,基频:1 =567 Hz 二次谐频:2 =1700 Hz 三次谐频:3 =2833 Hz 四次谐频:4 =3967 Hz 五次谐频:5 =5100 Hz 10-26 一平面简谐波的频率为500 Hz,在空气(ρ =1.3 kg·m -3 )中以u =340 m·s-1 的速度传播,到达人耳时,振幅约为A =1.0 ×10 -6 m.试求波在耳中的平均能量密度和声强. 解 波在耳中的平均能量密度 声强就是声波的能流密度,即 这个声强略大于繁忙街道上的噪声,使人耳已感到不适应.一般正常谈话的声强约1.0×10-6W·m-2 左右. 10-27  面积为1.0 m2 的窗户开向街道,街中噪声在窗口的声强级为80dB.问有多少“声功率”传入窗内? 分析 首先要理解声强、声强级、声功率的物理意义,并了解它们之间的相互关系.声强是声波的能流密度I,而声强级L 是描述介质中不同声波强弱的物理量.它们之间的关系为L =lg(I /I0 ),其中I0 =1.0 ×10-12 W·m-2为规定声强.L 的单位是贝尔(B),但常用的单位是分贝(dB),且1 B =10 dB.声功率是单位时间内声波通过某面积传递的能量,由于窗户上各处的I 相同,故有 =IS. 解 根据分析,由L =lg(I /I0 )可得声强为 I =10L I0 则传入窗户的声功率为 =IS =10L I0 S =1.0 ×10-4  W 10-28 若在同一介质中传播的,频率分别为1200 Hz 和400 Hz 的两声波有相同的振幅.求:(1) 它们的强度之比;(2) 两声波的声强级差. 解 (1) 因声强,则两声波声强之比 (2) 因声强级L =lg(I /I0 ),则两声波声强级差为 10-29 一警车以25 m·s-1 的速度在静止的空气中行驶,假设车上警笛的频率为800 Hz.求:(1) 静止站在路边的人听到警车驶近和离去时的警笛声波频率;(2) 如果警车追赶一辆速度为15m·s-1 的客车,则客车上人听到的警笛声波的频率是多少? (设空气中的声速u =330m·s-1 ) 分析 由于声源与观察者之间的相对运动而产生声多普勒效应,由多普勒频率公式可解得结果.在处理这类问题时,不仅要分清观察者相对介质(空气)是静止还是运动,同时也要分清声源的运动状态. 解 (1) 根据多普勒频率公式,当声源(警车)以速度vs =25 m·s-1  运动时,静止于路边的观察者所接收到的频率为 警车驶近观察者时,式中vs 前取“-”号,故有 警车驶离观察者时,式中vs 前取“+”号,故有 (2) 声源(警车)与客车上的观察者作同向运动时,观察者收到的频率为 10-30  一次军事演习中,有两艘潜艇在水中相向而行,甲的速度为50.0 km·h-1 ,乙的速度为km·h-1 ,如图所示.甲潜艇发出一个1.0×103 Hz的声音信号,设声波在水中的传播速度为5.47×103 km·h-1 ,试求(1) 乙潜艇接收到的信号频率;(2) 甲潜艇接收到的从乙潜艇反射回来的信号频率. 分析 (1) 甲潜艇是声源,发出信号频率为,乙潜艇是观察者,两者相向运动,利用多普勒频率公式,即可求得乙潜艇接收到的信号频率′.(2) 要求甲潜艇接收到的乙潜艇的信号频率,可将乙潜艇看成是声源,它发出的信号频率是′,将甲潜艇看成是观察者,两者相向运动,同样利用多普勒频率公式,可求出甲潜艇接收到的信号频率″. 题10-30 图 解 由题已知v1 =50.0 km·h-1 , v2 =70.0 km·h-1  , u =5.47 ×103 km·h-1 ,v =1000 Hz,由分析可知: (1)                      (2)                        *10-31 一广播电台的辐射功率是10 kW,假定辐射场均匀分布在以电台为中心的半球面上.(1) 求距离电台为r =10 kW 处的坡印廷矢量的平均值;(2) 若在上述距离处的电磁波可看作平面波,求该处的电场强度和磁场强度的振幅. 分析 坡印廷矢量是电磁波的能流密度矢量,它是随时间作周期性变化的.求其平均值,也就是指在一周期内的平均值.在忽略电磁波传播过程中的能量损耗时,按题意,波源的辐射功率就应等于单位时间通过半球面(面积A =2πr2 )的电磁波能量,即P =S·A,而平均能流密度值S =EH.另外,由电磁波的性质可知,E 与H 垂直,相位相同,且有关系式.因此,平面电磁波的坡印廷矢量大小的平均值可表示为,由此可求电场强度振幅 和磁场强度振幅 . 解 (1) 因为辐射场分布在半球面上,则坡印廷矢量的平均值为 (2) 根据分析, ,则磁场强度和电场强度的振幅分别为 *10-32   真空中有一平面电磁波的电场表达式如下: .求:(1) 波长,频率;(2) 该电磁 波的传播方向;(3) 磁场强度的大小和方向;(4) 坡印廷矢量. 分析 根据电磁波的特性,电场强度E 和磁场强度H 均垂直于波的传播方向.而E 和H 又互相垂直且同相位,E ×H 的方向为波速u 的方向.在数值上有关系成立.因此由题中给出的电场表达式可以求磁场表达式,而坡印廷矢量可由公式S =E ×H 求出. 解 (1) 由电场表达式可知,角频率ω=2π×108 s-1 ,波速u 等于光速c,则电磁波的波长和频率分别为 (2) 由电场表达式看出,电磁波沿x 轴正方向传播,E 矢量是在Oxy 平面内 偏振的. (3) 磁场强度表达式: H 矢量在Oxz 平面内偏振. (4) 第十一章 光  学 11-1 在双缝干涉实验中,若单色光源S 到两缝S1 、S2 距离相等,则观察屏 上中央明条纹位于图中O 处,现将光源S 向下移动到图中的S′位置,则(  ) (A) 中央明纹向上移动,且条纹间距增大 (B) 中央明纹向上移动,且条纹间距不变 (C) 中央明纹向下移动,且条纹间距增大 (D) 中央明纹向下移动,且条纹间距不变 分析与解 由S 发出的光到达S1 、S2 的光程相同,它们传到屏上中央O 处,光程差Δ=0,形成明纹.当光源由S 移到S′时,由S′到达狭缝S1 和S2 的两束光产生了光程差.为了保持原中央明纹处的光程差为0,它会向上移到图中O′处.使得由S′沿S1 、S2 狭缝传到O′处的光程差仍为0.而屏上各级条纹位置只是向上平移,因此条纹间距不变.因此正确答案为(B). 题11-1 图 11-2 如图所示,折射率为n2 ,厚度为e 的透明介质薄膜的上方和下方的透明介质的折射率分别为n1 和n3,且n1 <n2 ,n2 >n3 ,若用波长为λ的单色平行光垂直入射到该薄膜上,则从薄膜上、下两表面反射的光束的光程差是(  ) 题11-2 图 分析与解 由于n1 <n2 ,n2 >n3 ,因此在上表面的反射光有半波损失,下表面的反射光没有半波损失,故它们的光程差,这里λ是光在真空中的波长.因此正确答案为(B). 11-3 如图(a)所示,两个直径有微小差别的彼此平行的滚柱之间的距离为L,夹在两块平面晶体的中间,形成空气劈形膜,当单色光垂直入射时,产生等厚干涉条纹,如果滚柱之间的距离L 变小,则在L 范围内干涉条纹的(  ) (A) 数目减小,间距变大    (B) 数目减小,间距不变 (C) 数目不变,间距变小        (D) 数目增加,间距变小 题11-3图 分析与解 图(a)装置形成的劈尖等效图如图(b)所示.图中 d为两滚柱的直径差,b 为两相邻明(或暗)条纹间距.因为d 不变,当L 变小时,θ 变大,L′、b均变小.由图可得,因此条纹总数,因为d和λn 不变,所以N 不变.正确答案为(C) 11-4 在单缝夫琅禾费衍射实验中,波长为λ的单色光垂直入射在宽度为3λ的单缝上,对应于衍射角为30°的方向,单缝处波阵面可分成的半波带数目为(  ) (A) 2 个   (B) 3 个   (C) 4 个   (D) 6 个 分析与解 根据单缝衍射公式 因此第k 级暗纹对应的单缝波阵面被分成2k 个半波带,第k 级明纹对应的单缝波阵面被分成2k +1 个半波带.由题意,即对应第1 级明纹,单缝分成3 个半波带.正确答案为(B). 11-5 波长λ=550 nm 的单色光垂直入射于光栅常数d =1.0 ×10-4 cm 的光栅上,可能观察到的光谱线的最大级次为(  ) (A) 4     (B) 3     (C) 2     (D) 1 分析与解 由光栅方程,可能观察到的最大级次为 即只能看到第1 级明纹,答案为(D). 11-6  三个偏振片P1 、P2 与P3 堆叠在一起,P1 与P3的偏振化方向相互垂直,P2与P1 的偏振化方向间的夹角为45°,强度为I0 的自然光入射于偏振片P1 ,并依次透过偏振片P1 、P2与P3 ,则通过三个偏振片后的光强为(  ) (A) I0/16   (B) 3I0/8   (C) I0/8   (D) I0/4 分析与解 自然光透过偏振片后光强为I1 =I0/2.由于P1 和P2 的偏振化方向成45°,所以偏振光透过P2 后光强由马吕斯定律得.而P2和P3 的偏振化方向也成45°,则透过P3 后光强变为.故答案为(C). 11-7 一束自然光自空气射向一块平板玻璃,如图所示,设入射角等于布儒斯特角iB ,则在界面2 的反射光(  ) (A) 是自然光 (B) 是线偏振光且光矢量的振动方向垂直于入射面 (C) 是线偏振光且光矢量的振动方向平行于入射面 (D) 是部分偏振光 题11-7 图 分析与解 由几何光学知识可知,在界面2 处反射光与折射光仍然垂直,因此光在界面2 处的入射角也是布儒斯特角,根据布儒斯特定律,反射光是线偏振光且光振动方向垂直于入射面.答案为(B). 11-8 在双缝干涉实验中,两缝间距为0.30 mm,用单色光垂直照射双缝,在离缝1.20m 的屏上测得中央明纹一侧第5条暗纹与另一侧第5条暗纹间的距离为22.78 mm.问所用光的波长为多少,是什么颜色的光? 分析与解 在双缝干涉中,屏上暗纹位置由 决定,式中d′为双缝到屏的距离,d为双缝间距.所谓第5 条暗纹是指对应k =4 的那一级暗纹.由于条纹对称,该暗纹到中央明纹中心的距离,那么由暗纹公式即可求得波长λ.此外,因双缝干涉是等间距的,故也可用条纹间距公式求入射光波长.应注意两个第5 条暗纹之间所包含的相邻条纹间隔数为9(不是10,为什么?),故。 解1 屏上暗纹的位置,把以及d、d′值代入,可得λ=632.8 nm,为红光. 解2 屏上相邻暗纹(或明纹)间距,把,以及d、d′值代入,可得λ=632.8 nm. 11-9 在双缝干涉实验中,用波长λ=546.1 nm 的单色光照射,双缝与屏的距离d′=300mm.测得中央明纹两侧的两个第五级明条纹的间距为12.2mm,求双缝间的距离. 分析 双缝干涉在屏上形成的条纹是上下对称且等间隔的.如果设两明纹间隔为Δx,则由中央明纹两侧第五级明纹间距x5 -x-5 =10Δx 可求出Δx.再由公式Δx =d′λ/d 即可求出双缝间距d. 解 根据分析:Δx =(x5 -x-5)/10 =1.22×10-3 m 双缝间距:              d =d′λ/Δx =1.34 ×10-4 m 11-10 一个微波发射器置于岸上,离水面高度为d,对岸在离水面h 高度处放置一接收器,水面宽度为D,且,如图所示.发射器向对面发射波长为λ的微波,且λ>d,求接收器测到极大值时,至少离地多高? 分析 由发射器直接发射的微波与经水面反射后的微波相遇可互相干涉,这种干涉与劳埃德镜实验完全相同.形成的干涉结果与缝距为2d,缝屏间距为D 的双缝干涉相似,如图(b)所示,但要注意的是和劳埃德镜实验一样,由于从水面上反射的光存在半波损失,使得两束光在屏上相遇产生的光程差为,而不是. 题11-10 图 解 由分析可知,接收到的信号为极大值时,应满足 取k =1 时,得. 11-11 如图所示,由光源S 发出的λ=600 nm 的单色光,自空气射入折射率n=1.23的一层透明物质,再射入空气.若透明物质的厚度为d =1.0 cm,入射角θ =30°,且SA =BC =5.0cm,求:(1) 折射角θ1 为多少? (2) 此单色光在这层透明物质里的频率、速度和波长各为多少? (3) S 到C 的几何路程为多少?光程又为多少? 解 (1) 由折射定律可得 (2) 单色光在透明介质中的速度vn ,波长λn 和频率ν 分别为 (3) S 到C 的几何路程为 S 到C 的光程为 题11-11 图 11-12 一双缝装置的一个缝被折射率为1.40的薄玻璃片所遮盖,另一个缝被折射率为1.70 的薄玻璃片所遮盖.在玻璃片插入以后,屏上原来中央极大的所在点,现变为第五级明纹.假定λ=480nm,且两玻璃片厚度均为d,求d 值. 题11-12图 分析 本题是干涉现象在工程测量中的一个具体应用,它可以用来测量透明介质薄片的微小厚度或折射率.在不加介质片之前,两相干光均在空气中传播,它们到达屏上任一点P 的光程差由其几何路程差决定,对于点O,光程差Δ=0,故点O 处为中央明纹,其余条纹相对点O 对称分布.而在插入介质片后,虽然两相干光在两介质薄片中的几何路程相同,但光程却不同,对于点O,Δ≠0,故点O 不再是中央明纹,整个条纹发生平移.这时,干涉条纹空间分布的变化完全取决于光程差的变化.因此,对于屏上某点P(明纹或暗纹位置),只要计算出 插入介质片前后光程差的变化,即可知道其干涉条纹的变化情况. 插入介质前的光程差Δ1 =r1 -r 2 =k1 λ(对应k1 级明纹),插入介质后的光程差Δ2 =[(n1-1)d +r1 ]-[(n2 -1)d +r2 ]=k2 λ(对应k2 级明纹).光程差的变化量为 Δ2 -Δ1 =(n2 -n1 )d =(k2 -k1 )λ 式中(k2 -k1 )可以理解为移过点P 的条纹数(本题为5).因此,对于这类问题,求解光程差的变化量是解题的关键. 解 由上述分析可知,两介质片插入前后,对于原中央明纹所在点O,有 将有关数据代入可得 11-13 白光垂直照射到空气中一厚度为380 nm 的肥皂膜上.设肥皂的折射率为1.32.试问该膜的正面呈现什么颜色? 背面呈现什么颜色? 分析 这是薄膜干涉问题,求正面呈现的颜色就是在反射光中求因干涉增强光的波长(在可见光范围),求背面呈现的颜色就是在透射光中求干涉增强(即反射减弱)光的波长. 解 根据分析对反射光加强,有 在可见光范围,k =2 时,(红光) k =3 时,(紫光) 故正面呈红紫色.同理,对透射光加强,有 2ne =kλ (k =1,2,…) 在可见光范围仅有k=2 时,λ=501.6 nm(绿光).即背面呈绿色. 11-14 在折射率n3 =1.52 的照相机镜头表面涂有一层折射率n2 =1.38的MgF2 增透膜,若此膜仅适用于波长λ=550nm的光,则此膜的最小厚度为多少? 分析 在薄膜干涉中,膜的材料及厚度都将对两反射光(或两透射光)的光程差产生影响,从而可使某些波长的光在反射(或透射)中得到加强或减弱,这种选择性使薄膜干涉在工程技术上有很多应用.本题所述的增透膜,就是希望波长λ=550nm的光在透射中得到加强,从而得到所希望的照相效果(因感光底片对此波长附近的光最为敏感).具体求解时应注意在d>0的前提下,k 取最小的允许值. 解1 因干涉的互补性,波长为550nm 的光在透射中得到加强,则在反射中一定减弱,两反射光的光程差Δ2 =2n2 d,由干涉相消条件 ,得 取k =0,则dmin =99.6nm. 解2 由于空气的折射率n1 =1,且有n1 <n2 <n3 ,则对透射光而言,两相干光的光程差,由干涉加强条件Δ1 =kλ,得 取k =1,则膜的最小厚度dmin =99.6nm. 11-15 利用空气劈尖测细丝直径.如图所示,已知λ=589.3 nm,L =2.888 ×10-2m,测得30 条条纹的总宽度为4.259 ×10-3 m,求细丝直径d. 分析 在应用劈尖干涉公式 时,应注意相邻条纹的间距b 是N 条条纹的宽度Δx 除以(N -1).对空气劈尖n =1. 解 由分析知,相邻条纹间距,则细丝直径为 题11-15 图 11-16 集成光学中的楔形薄膜耦合器原理如图所示.沉积在玻璃衬底上的是氧化钽()薄膜,其楔形端从A 到B 厚度逐渐减小为零.为测定薄膜的厚度,用波长λ=632.8nm 的 激光垂直照射,观察到薄膜楔形端共出现11 条暗纹,且A 处对应一条暗纹,试求氧化钽薄膜的厚度.( 对632.8 nm激光的折射率为2.21) 题11-16 图 分析 置于玻璃上的薄膜AB 段形成劈尖,求薄膜厚度就是求该劈尖在A点处的厚度.由于 对激光的折射率大于玻璃,故从该劈尖上表面反射的光有半波损失,而下表面没有,因而两反射光光程差为Δ=2ne +λ/2.由反射光暗纹公式2ne k +λ/2 =(2k +1)λ/2,k =0,1,2,3,…,可以求厚度ek .又因为AB 中共有11 条暗纹(因半波损失B 端也为暗纹),则k 取10即得薄膜厚度. 解 根据分析,有 2ne k +λ/2 =(2k +1)λ/2 (k =0,1,2,3,…) 取k =10,得薄膜厚度e10 =10λ/2n =1.4 ×10-6m. 11-17 如图(a)所示,将符合标准的轴承钢珠a、b 和待测钢珠c 一起放在两块平板玻璃之间,若垂直入射光的波长λ=580 nm,问钢珠c 的直径比标准小多少? 如果距离d 不同,对检测结果有何影响? 分析 很显然,如钢珠c 与标准件a、b 相同,则呈现厚度相同的薄膜干涉;如钢珠与标准件不同,则为劈尖干涉.后者有等厚干涉条纹出现,a 与c 之间的条纹分布如图(b)所示.由于相邻条纹的厚度差Δd =λ/2n.而空气的折射率n≈1,则两钢珠之间的直径差,式中N 为a 与c 之间的条纹间隔数目(注:条纹数目较多时,也可用条纹数目作近似计算),由图(a)知N 约为. 改变钢珠间的距离d,将钢珠c 移至c′处,如图(c)所示,a 与c′之间条纹数并未改变,但由于相邻条纹间距变小,从而影响观测. 题11-17 图 解 钢珠c 和a、b 的直径不同,则两平板玻璃形成空气劈.由分析得,钢珠c的直径与标准件直径相差 当距离d 稍微改变时,a、b 与c 之间条纹数目未变,故不影响检验结果. 11-18 折射率为1.60 的两块标准平面玻璃板之间形成一个劈形膜(劈尖角θ 很小).用波长λ=600nm 的单色光垂直入射,产生等厚干涉条纹.假如在劈形膜内充满n =1.40 的液体时的相邻明纹间距比劈形膜内是空气时的间距缩小Δl =0.5 mm,那么劈尖角θ 应是多少? 分析 劈尖干涉中相邻条纹的间距l≈λ/2nθ,其中θ 为劈尖角,n 是劈尖内介质折射率.由于前后两次劈形膜内介质不同,因而l 不同.则利用l≈λ/2nθ和题给条件可求出θ. 解 劈形膜内为空气时, 劈形膜内为液体时, 则由,得 11-19 如图所示的干涉膨胀仪,已知样品的平均高度为3.0 ×10-2m,用λ=589.3 nm的单色光垂直照射.当温度由17 ℃上升至30 ℃时,看到有20 条条纹移过,问样品的热膨胀系数为多少? 题11-19 图 分析 温度升高ΔT =T2 -T1 后,样品因受热膨胀,其高度l 的增加量Δl =lαΔT.由于样品表面上移,使在倾角θ 不变的情况下,样品与平板玻璃间的空气劈的整体厚度减小.根据等厚干涉原理,干涉条纹将整体向棱边平移,则原k 级条纹从a 移至a′处,如图(b)所示,移过某一固定观察点的条纹数目N 与Δl 的关系为,由上述关系可得出热膨胀系数α. 解 由题意知,移动的条纹数N =20,从分析可得 则热膨胀系数                  11 -20 在利用牛顿环测未知单色光波长的实验中,当用已知波长为589.3 nm的钠黄光垂直照射时,测得第一和第四暗环的距离为Δr =4.00 ×10-3 m;当用波长未知的单色光垂直照射时,测得第一和第四暗环的距离为Δr′=3.85 ×10-3 m,求该单色光的波长. 分析 牛顿环装置产生的干涉暗环半径,其中k =0,1,2…,k =0,对应牛顿环中心的暗斑,k=1 和k =4 则对应第一和第四暗环,由它们之间的间距,可知,据此可按题中的测量方法求出未知波长λ′. 解 根据分析有 故未知光波长                      λ′=546 nm 11-21 在牛顿环实验中,当透镜与玻璃之间充以某种液体时,第10 个亮环的直径由1.40×10-2 m变为1.27 ×10-2 m,试求这种液体的折射率. 分析 当透镜与平板玻璃间充满某种液体(n2 >1),且满足n1 >n2 ,n2 <n3或n1 <n2 ,n2 >n3 时,在厚度为d 的地方,两相干光的光程差为.由此可推导出牛顿环暗环半径和明环半径,这里明、暗环半径和充入的介质折射率n2 有关.有兴趣的读者可自行推导.必须指出,在牛顿环中,若介质不均匀或分析的是透射光而不是反射光,那么关于暗环、明环半径的公式与教材中的公式是不同的,不能随意套用. 解 当透镜与玻璃之间为空气时,k 级明纹的直径为 当透镜与玻璃之间为液体时,k 级明纹的直径为 解上述两式得 11 -22 如图所示,折射率n2 =1.2 的油滴落在n3 =1.50 的平板玻璃上,形成一上表面近似于球面的油膜,测得油膜中心最高处的高度dm =1.1 μm,用λ=600 nm 的单色光垂直照射油膜,求(1) 油膜周边是暗环还是明环? (2) 整个油膜可看到几个完整的暗环? 题11-22 图 分析 本题也是一种牛顿环干涉现象,由于n1 <n2 <n3 ,故油膜上任一点处两反射相干光的光程差Δ=2n2d.(1) 令d =0,由干涉加强或减弱条件即可判断油膜周边是明环.(2) 由2n2d =(2k +1)λ/2,且令d =dm 可求得油膜上暗环的最高级次(取整),从而判断油膜上完整暗环的数目. 解 (1) 根据分析,由 油膜周边处d =0,即Δ=0 符合干涉加强条件,故油膜周边是明环. (2) 油膜上任一暗环处满足 令d =dm ,解得k =3.9,可知油膜上暗环的最高级次为3,故油膜上出现的完整暗环共有4 个,即k =0,1,2,3. 11-23 把折射率n =1.40 的薄膜放入迈克耳孙干涉仪的一臂,如果由此产生了7.0 条条纹的移动,求膜厚.设入射光的波长为589 nm. 分析 迈克耳孙干涉仪中的干涉现象可以等效为薄膜干涉(两平面镜相互垂直)和劈尖干涉(两平面镜不垂直)两种情况,本题属于后一种情况.在干涉仪一臂中插入介质片后,两束相干光的光程差改变了,相当于在观察者视野内的空气劈尖的厚度改变了,从而引起干涉条纹的移动. 解 插入厚度为d 的介质片后,两相干光光程差的改变量为2(n -1)d,从而引起N 条条纹的移动,根据劈尖干涉加强的条件,有2(n -1)d=Nλ,得d= Nλ 11-24 如图所示,狭缝的宽度b =0.60 mm,透镜焦距f =0.40m,有一与狭缝平行的屏放置在透镜焦平面处.若以单色平行光垂直照射狭缝,则在屏上离点O 为x =1.4 mm处的点P,看到的是衍射明条纹.试求:(1) 该入射光的波长;(2) 点P条纹的级数;(3) 从点P 看来对该光波而言,狭缝处的波阵面可作半波带的数目. 分析 单缝衍射中的明纹条件为,在观察点P 确定(即φ确定)后,由于k 只能取整数值,故满足上式的λ只可取若干不连续的值,对照可见光的波长范围可确定入射光波长的取值.此外,如点P 处的明纹级次为k,则狭缝处的波阵面可以划分的半波带数目为(2k +1),它们都与观察点P 有关,φ越大,可以划分的半波带数目也越大. 解 (1) 透镜到屏的距离为d,由于d >>b,对点P 而言,有.根据单缝衍射明纹条件,有 将b、d(d≈f)、x 的值代入,并考虑可见光波长的上、下限值,有 因k 只能取整数值,故在可见光范围内只允许有k =4 和k =3,它们所对应的入 射光波长分别为λ2 =466.7 nm和λ1 =600 nm. (2) 点P 的条纹级次随入射光波长而异,当λ1 =600 nm时,k =3;当λ2 =466.7 nm时,k =4. (3) 当λ1 =600 nm 时,k =3,半波带数目为(2k +1) =7;当λ2 =466.7 nm 时,k =4,半波带数目为9. 题11-24 图 11-25 单缝的宽度b =0.40 mm,以波长λ=589 nm 的单色光垂直照射,设透镜的焦距f =1.0 m.求:(1) 第一级暗纹距中心的距离;(2) 第二级明纹距 中心的距离;*(3) 如单色光以入射角i =30°斜射到单缝上,则上述结果有何变动. 题11-25图 分析 对于问题(3)单色光倾斜入射单缝的情况,在入射光到达单缝时,其上下两列边界光线之间已存在光程差(若为光栅,则为),对应等光程的中央主极大将移至点O′(此时φ=i =30°),屏上衍射条纹原有的对称性受到一定的破坏. 如图所示,对于点O′上方的条纹(此时入射光与衍射光位于法线两侧,且φ>i),满足 如令,可求得最大条纹级次km1 .对于点O 下方的条纹(此时入射光与衍射光位于法线同侧),满足 如令,可求得另一侧的最大条纹级次km2 .对于点O′与O 之间的条纹(此时入射光与衍射光位于法线两侧,但φ<i),满足 需要说明的是,点O′与O之间的条纹与点O下方的条纹属于中央主极大同一侧的各级条纹,不同的是前者k 值较小,后者k 值较大,且k 值在点O附近连续变化. 解 (1) 由单缝衍射的暗纹条件,得,则第一级(k =1)暗纹距中心的距离为 (2) 由明纹条件,得,则第二级(k =2)明纹距中心的距离为 在上述计算中,由于k 取值较小,即φ较小,故.如k 取值较大,则应严格计算.   *(3) 斜入射时,中央主极大移至点O′,先计算点O′上方条纹的位置:对于第 一级暗纹,有,该暗纹距中心的距离 对于第二级明纹,有,该明纹距中心的距离 再计算O′点下方条纹的位置(由于所求k 值较小,其条纹应在O′与O 之间):对于第一级暗纹,有,该暗纹距中心的距离 对于第二级明纹,有,该明纹距中心的距离 讨论 斜入射时,中央主极大移至点O′(此时φ=i =30°),它距中心点O的距离为,由上述计算数据可知,此时衍射条纹不但相对点O 不对称,而且相对中央主极大的点O′也不再严格对称了. 11-26 一单色平行光垂直照射于一单缝,若其第三条明纹位置正好和波长为600 nm的单色光垂直入射时的第二级明纹的位置一样,求前一种单色光的波长. 分析 采用比较法来确定波长.对应于同一观察点,两次衍射的光程差相同,由于衍射明纹条件,故有,在两明纹级次和其中一种波长已知的情况下,即可求出另一种未知波长. 解 根据分析,将代入( ,得 11-27  已知单缝宽度b =1.0 ×10-4 m,透镜焦距f =0.5 m,用λ1 =400 nm和λ2 =760 nm的单色平行光分别垂直照射,求这两种光的第一级明纹离屏中心的距离,以及这两条明纹之间的距离.若用每厘米刻有1000条刻线的光栅代替这个单缝,则这两种单色光的第一级明纹分别距屏中心多远? 这两条明纹之间的距离又是多少? 分析 用含有两种不同波长的混合光照射单缝或光栅,每种波长可在屏上独立地产生自己的一组衍射条纹,屏上最终显示出两组衍射条纹的混合图样.因而本题可根据单缝(或光栅)衍射公式分别计算两种波长的k 级条纹的位置x1和x2 ,并算出其条纹间距Δx =x2 -x1 .通过计算可以发现,使用光栅后,条纹将远离屏中心,条纹间距也变大,这是光栅的特点之一. 解 (1) 当光垂直照射单缝时,屏上第k 级明纹的位置 当λ1 =400 nm 和k =1 时,  x1 =3.0 ×10-3 m 当λ2 =760 nm 和k =1 时,  x2 =5.7 ×10-3 m 其条纹间距    Δx =x2 -x1 =2.7 ×10-3 m (2) 当光垂直照射光栅时,屏上第k 级明纹的位置为 而光栅常数                    当λ1 =400 nm 和k =1 时,  x1 =2.0 ×10-3 m 当λ2 =760 nm 和k =1 时,  x2 =3.8 ×10-3 m 其条纹间距                 11-28 迎面而来的一辆汽车的两车头灯相距为1.0 m,问在汽车离人多远时,它们刚能为人眼所分辨? 设瞳孔直径为3.0 m m,光在空气中的波长λ=500 nm. 分析 两物体能否被分辨,取决于两物对光学仪器通光孔(包括人眼)的张角θ 和光学仪器的最小分辨角θ0 的关系.当θ≥θ0 时能分辨,其中θ=θ0 为恰能分辨.在本题中为一定值,而,式中l 为两灯间距,d 为人与车之间的距离.d 越大或l 越小,θ 就越小,当θ <θ0  时两灯就不能被分辨,这与我们的生活经验相符合. 解 当θ =θ0时, ,此时,人与车之间的距离为 11-29 老鹰眼睛的瞳孔直径约为6 mm,问其最多飞翔多高时可看清地面上身长为5cm的小鼠? 设光在空气中的波长为600 nm. 解 根据上题的分析:θ0 =1.22λ/D.这里D 是鹰的瞳孔直径.而θ =L/h,其中L 为小鼠的身长, h 为老鹰飞翔的高度.恰好看清时θ =θ0, 则由L/h =1.22λ/D,得飞翔高度:h =LD/(1.22λ) =409.8 m. 11-30 一束平行光垂直入射到某个光栅上,该光束中包含有两种波长的光:λ1 =440 nm 和λ2 =660 nm.实验发现,两种波长的谱线(不计中央明纹)第二次重合于衍射角φ=60°的方向上,求此光栅的光栅常数. 分析 根据光栅衍射方程,两种不同波长的谱线,除k =0 中央明纹外,同级明纹在屏上位置是不同的,如果重合,应是它们对应不同级次的明纹在相同衍射角方向上重合.故由dsin φ=kλ1 =k′λ2 可求解本题. 解 由分析可知        得                        上式表明第一次重合是λ1 的第3 级明纹与λ2 的第2级明纹重合,第二次重合是λ1 的第6 级明纹与λ2 的第4级明纹重合.此时,k =6,k′=4,φ=60°,则光栅常数 11-31 用一个1.0mm 内有500 条刻痕的平面透射光栅观察钠光谱(λ=589nm),设透镜焦距f =1.00 m.问:(1) 光线垂直入射时,最多能看到第几级光谱;*(2) 光线以入射角30°入射时,最多能看到第几级光谱;(3) 若用白光垂直照射光栅,求第一级光谱的线宽度. 分析 (1) 首先确定光栅常数,式中N 为刻痕数,然后由光线垂直照射光栅时的衍射条件,即可解得结果. (2) 如同光线倾斜入射单缝一样, 此时光栅衍射的明纹条件改变为(详见题11-25 的分析),由于两侧条纹不再对称,令,可求得km1 和km2 两个值,其中一个比垂直入射时的km 值小,另一个比km 值大,因而,在其他条件不变的情况下,倾斜入射时可以观察到较高级次的条纹. (3) 用白光照射光栅,除中央明纹仍为白光外,其余处出现一系列光谱带,称为光栅光谱.每个光谱带是由同一级次不同波长的明纹依次排列而成.所谓第一级光谱的线宽度是指入射光中最小波长(取)和最大波长(取)的第一级明纹在屏上的间距,其余波长的第一级明纹均出现在此范围内.需要指出的是,对于较高级次的光谱会出现相邻光谱间的交错重叠的现象. 解 (1) 光波垂直入射时, 光栅衍射明纹的条件为,令,可得 取整数km =3,即最多能看到第3级光谱. (2) 倾斜入射时,光栅明纹的条件为 令,可求得位于中央主极大两侧,能观察到条纹的最大km 值分别为km1 =5和km2 =1(已取整数值).故在法线两侧能观察到的最大级次分别为五级和一级. (3) 白光的波长范围为400 nm ~760 nm,用白光垂直照射时,由可得第一级(k =1)光谱在屏上的位置.对应于λ1 =400 nm 和λ2 =760 nm 的明纹的衍射角为,利用可得明纹的位置为 则第一级光谱的线宽度为 11-32 波长为600 nm 的单色光垂直入射在一光栅上,第二级主极大出现在处,第四级缺级.试问(1) 光栅上相邻两缝的间距是多少?(2) 光栅上狭缝的宽度有多大? (3) 在-90°<φ<90°范围内,实际呈现的全部级数. 分析 (1) 利用光栅方程,即可由题给条件求出光栅常数d(即两相邻缝的间距).(2) 光栅衍射是多缝干涉的结果,也可看成是光透过许多平行的单缝衍射的结果.缺级就是按光栅方程计算屏上某些应出现明纹的位置,按各个单缝衍射计算恰是出现暗纹的位置.因此可以利用光栅方程和单缝衍射暗纹公式计算缝宽和屏上缺级的情况,从而求出屏上条纹总数. 解 (1) 由题已知k =2 时,,则由分析可得光栅常数: (2) 由分析知缺级条件 则(b +b′)/b =k/k′=m,k =m k′,即m k′级明纹缺级. 由题意k =4 缺级,即(b +b′)/b =4/k′ 当k′=1 时,m =4,,即±4, ±8, ±12,…级缺级.(符合题意) 当k′=2 时,m =2,第±2, ±4, ±6,…级缺级.(第二级已存在,不符合题意,舍 去) 当k′=3 时,,第±4, ±8, ±12,…级缺级.(符合题意) 当k′=4 时,m =1,第±1, ±2, ±3, ±4,…级全部缺级.(不符合题意,舍去) 因此,狭缝宽度b 为1.5 μm 或者4.5μm,而缺级只发生在±4, ±8, ±12,…级. (3) 由光栅方程,可知屏上呈现条纹最高级次应满足,故考虑到缺级,实际屏上呈现的级数为:0, ±1, ±2, ±3,±5, ±6, ±7, ±9,共15 条. 11-33 以波长为0.11 nm 的X 射线照射岩盐晶体,实验测得X 射线与晶面夹角为11.5°时获得第一级反射极大.(1) 岩盐晶体原子平面之间的间距d为多大? (2) 如以另一束待测X 射线照射,测得X 射线与晶面夹角为17.5°时获得第一级反射光极大,求该X 射线的波长. 分析 X 射线入射到晶体上时,干涉加强条件为2dsin θ =kλ(k =0,1,2,…)式中d 为晶格常数,即晶体内原子平面之间的间距(如图). 解 (1) 由布拉格公式 第一级反射极大,即k =1.因此,得   (2) 同理,由2dsinθ2 =kλ2 ,取k =1,得 题11-33图 11-34 测得一池静水的表面反射出来的太阳光是线偏振光,求此时太阳处在地平线的多大仰角处? (水的折射率为1.33) 题11-34 图 分析 设太阳光(自然光)以入射角i 入射到水面,则所求仰角.当反射光起偏时,根据布儒斯特定律,有(其中n1 为空气的折射率,n2 为水的折射率). 解 根据以上分析,有 则                            11-35 使自然光通过两个偏振化方向相交60°的偏振片,透射光强为I1 ,今在这两个偏振片之间插入另一偏振片,它的方向与前两个偏振片均成30°角,则透射光强为多少? 分析 设入射自然光强为I0 ,偏振片I对入射的自然光起起偏作用,透射的偏振光光强恒为,而偏振片Ⅱ对入射的偏振光起检偏作用,此时透射与入射的偏振光强满足马吕斯定律.若偏振片Ⅲ插入两块偏振片之间,则偏振片Ⅱ、Ⅲ均起检偏作用,故透射光强必须两次应用马吕斯定律方能求出. 解 根据以上分析,入射光通过偏振片Ⅰ和Ⅱ后,透射光强为 插入偏振片Ⅲ后,其透射光强为 两式相比可得  11-36 一束光是自然光和线偏振光的混合,当它通过一偏振片时,发现透射光的强度取决于偏振片的取向,其强度可以变化5 倍,求入射光中两种光的强度各占总入射光强度的几分之几. 分析 偏振片的旋转,仅对入射的混合光中的线偏振光部分有影响,在偏振片旋转一周的过程中,当偏振光的振动方向平行于偏振片的偏振化方向时,透射光强最大;而相互垂直时,透射光强最小.分别计算最大透射光强Imax 和最小透射光强Imin ,按题意用相比的方法即能求解. 解 设入射混合光强为I,其中线偏振光强为xI,自然光强为(1-x)I.按题意旋转偏振片,则有最大透射光强                    最小透射光强                          按题意,则有 解得                               x =2/3 即线偏振光占总入射光强的2/3,自然光占1/3.   *11-37 试分别计算用方解石晶体制成的对波长为λ1 =589.3nm 的钠黄光和波长为λ2 =546.1nm 的汞灯绿光的1/4波片的最小厚度为多少? 解 1/4 波片的最小厚度d 应满足,而方解石晶体中o 光和e光的折射率分别为n0 =1.658 和ne =1.486.故对λ1 和λ2 的1/4波片的最小厚度分别为   *11-38 在偏振化方向相互平行的两偏振片间,平行放置一片垂直于光轴切割的石英晶片.已知石英对钠黄光的旋光率为21.7(°)/mm。试问石英晶片的厚度为多大时,钠黄光不能通过第二个偏振片? 分析 按题意,第一个偏振片透射的线偏振光将沿光轴方向进入石英晶片.由于石英具有旋光性,光的振动方向将向右或向左旋转一个角度ψ.其大小与晶片厚度l 成正比,即ψ=αl,比例系数α为晶体的旋光率.很显然欲使纳黄光不能通过第二个偏振片、旋光角度ψ应90°. 解 根据分析,石英晶片的厚度为 *11-39 一块厚0.025 mm 的方解石晶片,其表面与光轴平行,放置在两正交的偏振片之间.已知第一个偏振片的偏振化方向与晶片的光轴成45°角.现用白光垂直入射第一个偏振片,问经第二个偏振片透射的可见光谱中,缺少了哪些波长? 假定双折射率no-ne =0.172 可看作常量. 题11-39 图 分析 本题为偏振光的干涉问题.按题意作图,经方解石晶体透射的两束线偏振光(指o光与e 光),其振动方向相互垂直,再经第二个偏振片的检偏作用后成为相干光.其相位差为 ,式中第一项为o光与e 光通过晶片时产生的相位差,第二项则 为两光的Ao 与Ae 在N2 (第二个偏振片的偏振化方向)上投影时有可能产生的附加相位差.由图知,本题应取π.在可见光范围内讨论,由干涉相消条件,可解得透射的可见光谱中少了哪些波长. 解 根据分析,有                           (1)                               (2) 解上述两式可得 在可见光范围(400 nm ~760 nm)内讨论,解得 很显然,在缺少了上述波长后,透射光不再是白光了. 讨论 若两偏振片的偏振化方向相互平行,情况会如何?
本文档为【大学物理 马文蔚 第五版 下册 第九章到第十一章课后答案】,请使用软件OFFICE或WPS软件打开。作品中的文字与图均可以修改和编辑, 图片更改请在作品中右键图片并更换,文字修改请直接点击文字进行修改,也可以新增和删除文档中的内容。
该文档来自用户分享,如有侵权行为请发邮件ishare@vip.sina.com联系网站客服,我们会及时删除。
[版权声明] 本站所有资料为用户分享产生,若发现您的权利被侵害,请联系客服邮件isharekefu@iask.cn,我们尽快处理。
本作品所展示的图片、画像、字体、音乐的版权可能需版权方额外授权,请谨慎使用。
网站提供的党政主题相关内容(国旗、国徽、党徽..)目的在于配合国家政策宣传,仅限个人学习分享使用,禁止用于任何广告和商用目的。
下载需要: 免费 已有0 人下载
最新资料
资料动态
专题动态
is_003124
暂无简介~
格式:doc
大小:1MB
软件:Word
页数:66
分类:生活休闲
上传时间:2017-09-19
浏览量:1179